Difference between revisions of "2019 AMC 10A Problems/Problem 24"

(Created page with "hey")
 
Line 1: Line 1:
hey
+
==Problem==
 +
 
 +
Let <math>p</math>, <math>q</math>, and <math>r</math> be the distinct roots of the polynomial <math>x^3 - 22x^2 + 80x - 67</math>. It is given that there exist real numbers <math>A</math>, <math>B</math>, and <math>C</math> such that <cmath>\dfrac{1}{s^3 - 22s^2 + 80s - 67} = \dfrac{A}{s-p} + \dfrac{B}{s-q} + \frac{C}{s-r}</cmath>for all <math>s\not\in\{p,q,r\}</math>. What is <math>\tfrac1A+\tfrac1B+\tfrac1C</math>?
 +
 
 +
<math>\textbf{(A) }243\qquad\textbf{(B) }244\qquad\textbf{(C) }245\qquad\textbf{(D) }246\qquad\textbf{(E) } 247</math>
 +
 
 +
==Solution==
 +
 
 +
==See Also==
 +
 
 +
{{AMC10 box|year=2019|ab=A|num-b=23|num-a=25}}
 +
{{MAA Notice}}

Revision as of 16:56, 9 February 2019

Problem

Let $p$, $q$, and $r$ be the distinct roots of the polynomial $x^3 - 22x^2 + 80x - 67$. It is given that there exist real numbers $A$, $B$, and $C$ such that \[\dfrac{1}{s^3 - 22s^2 + 80s - 67} = \dfrac{A}{s-p} + \dfrac{B}{s-q} + \frac{C}{s-r}\]for all $s\not\in\{p,q,r\}$. What is $\tfrac1A+\tfrac1B+\tfrac1C$?

$\textbf{(A) }243\qquad\textbf{(B) }244\qquad\textbf{(C) }245\qquad\textbf{(D) }246\qquad\textbf{(E) } 247$

Solution

See Also

2019 AMC 10A (ProblemsAnswer KeyResources)
Preceded by
Problem 23
Followed by
Problem 25
1 2 3 4 5 6 7 8 9 10 11 12 13 14 15 16 17 18 19 20 21 22 23 24 25
All AMC 10 Problems and Solutions

The problems on this page are copyrighted by the Mathematical Association of America's American Mathematics Competitions. AMC logo.png